smoke

This topic has expert replies
Legendary Member
Posts: 2326
Joined: Mon Jul 28, 2008 3:54 am
Thanked: 173 times
Followed by:2 members
GMAT Score:710

smoke

by gmatmachoman » Sun Dec 06, 2009 3:40 am
One year ago a local government initiated an antismoking advertising campaign in local newspapers which it financed by imposing a tax on cigarettes of 20 cents per pack One year later the number of people in the locality who smoke cigarettes had declined by 3 percent Clearly what was said in the advertisements had an effect although a small one on the number of people in the locality who smoke cigarettes.

Which one of the following, if true, most helps to strengthen argument?

(A) Residents of the locality have not increased their use of other tobacco products such as snuff and chewing tobacco since the campaign went into effect

(B) A substantial number of cigarette smokers in the locality who did not quit smoking during the campaign now smoke less than they did before it began

(C) Admissions to the local hospital for chronic respiratory ailments were down by 15 percent one year after the campaign began

(D) Merchants in the locality responded to the local tax by reducing the price at which they sold cigarettes by 20 cents per pack

(E) Smokers in the locality had incomes that on average were 25 percent lower than those of nonsmokers

Newbie | Next Rank: 10 Posts
Posts: 7
Joined: Mon Nov 16, 2009 1:01 pm
Thanked: 1 times

by salar_snake » Sun Dec 06, 2009 4:50 am
IMO D.

(D) Merchants in the locality responded to the local tax by reducing the price at which they sold cigarettes by 20 cents per pack


---> the 20 cents tax per pack was neutralized by price decrease ----> the only reason for the minor reduction in cigarettes smoking was "what was said in the advertisement"

Legendary Member
Posts: 1035
Joined: Wed Aug 27, 2008 10:56 pm
Thanked: 104 times
Followed by:1 members

by scoobydooby » Sun Dec 06, 2009 9:03 am
would go for B, all others are kind of out of scope. they do not address the effect on the cigarette smokers.

Legendary Member
Posts: 527
Joined: Mon Jun 02, 2008 9:14 am
Location: Atlanta
Thanked: 17 times

by pandeyvineet24 » Sun Dec 06, 2009 9:13 am
i would go for D. The conclusion says that advertisement made only a small impact even though effectively the price of the pack of cig. remained the same.
B uses an extreme word "substantial number", signifying a large impact.

Master | Next Rank: 500 Posts
Posts: 256
Joined: Mon Aug 10, 2009 6:31 pm
Thanked: 3 times

by gmatv09 » Sun Dec 06, 2009 9:13 am
the conclusion - "the ad had an effect on the cigarette smoking ..."

IMO B


D = provides a possible reason for the minor reduction in cigarette smoking

GMAT Instructor
Posts: 1302
Joined: Mon Oct 19, 2009 2:13 pm
Location: Toronto
Thanked: 539 times
Followed by:164 members
GMAT Score:800

by Testluv » Sun Dec 06, 2009 6:57 pm
The correct answer must be choice D.

This is another explain the phenomenon or cause of the effect type of argument.

The first and second sentences describe a phenomenon, while the third sentence advances an explanation for that phenomenon.

The phenomenon: Smoking declined.

The author's explanation: The ads (and not the taxes!) caused the decrease in smoking. ("Clearly what was said in the advertisements had an effect"...)

Whenever the author makes an explain-the-phenomenon argument, he is assuming that there are no other explanations.

The way we strengthen an argument is by finding an answer choice that backs up the assumption.

How do we back up the assumption that there are no other explanations?

The most common way it will happen on the GMAT is to look for an answer choice that negates an alternative explanation.

A clear competing alternative explanation from the stimulus is the tax itself. (That is, instead of the ads' content causing the decline in smoking, it could have been the tax-fuelled increase in the costs of the cigarettes that caused the decline in smoking).

So, we would be looking for any choice that says "the tax was not the reason the smoking went down".

Choice D is a perfect match to our prediction. It tells us that the merchants absorbed the cigarette tax; therefore, the consumers who buy the cigarettes did not suffer, did not incur, the tax increase.

The fact that they did not have to pay more for the cigarettes and that they still decreased their smoking strengthens the author's argument that what was said in the ads caused the smoking decline.

Choice B does not strengthen the argument that smoking decreased because of the ads' content. Instead, it simply affirms the fact that smoking did decrease. In other words, choice B does not give us any new information; we already knew from the passage that smoking declined. For, that was the very phenomenon about which the author was advancing an explanation.

Although this looks like an LSAT question, it is a very fair and even a good representation of how strengthening/weakening commonly works on the GMAT. Many of the strengthen/weaken questions on the GMAT are of the "explain the phenomenon" variety. The quicker you are able to spot the form, and the better you know how to analyze the form, the more efficiently and confidently and easiliy you can answer a question like this. Therefore, according to me, a great idea would be to go through the CR section of the OG, explicitly pulling out those stn/wkn questions that use this form.
Kaplan Teacher in Toronto

Legendary Member
Posts: 2326
Joined: Mon Jul 28, 2008 3:54 am
Thanked: 173 times
Followed by:2 members
GMAT Score:710

by gmatmachoman » Sun Dec 06, 2009 8:46 pm
TestLuv,

Once again such a wonderful post from you.

What makes me feel" mesmerised" is u r too good in deciphering the type of question be it LSAT or GMAT ones!!

Yeah this one was from LSAT....

Many Thx bro

User avatar
Master | Next Rank: 500 Posts
Posts: 166
Joined: Fri Apr 03, 2009 11:16 pm
Location: India

by Mayur Sand » Sun Dec 06, 2009 10:12 pm
what is the OA ?

GMAT Instructor
Posts: 1302
Joined: Mon Oct 19, 2009 2:13 pm
Location: Toronto
Thanked: 539 times
Followed by:164 members
GMAT Score:800

by Testluv » Sun Dec 06, 2009 10:21 pm
Mayur Sand wrote:what is the OA ?
Well, since the original poster reacted positively to my explanation, I think we can infer that the OA is the choice I thought was correct: choice D!
Kaplan Teacher in Toronto

User avatar
Master | Next Rank: 500 Posts
Posts: 434
Joined: Mon Jun 11, 2007 9:48 pm
Location: Bangalore
Thanked: 6 times
GMAT Score:600

by viju9162 » Mon Dec 07, 2009 1:18 am
amazing explanation Testluv!!! When can I think like you while solving CR questions :D :cry: :!: :| :?
"Native of" is used for a individual while "Native to" is used for a large group

GMAT Instructor
Posts: 1302
Joined: Mon Oct 19, 2009 2:13 pm
Location: Toronto
Thanked: 539 times
Followed by:164 members
GMAT Score:800

by Testluv » Mon Dec 07, 2009 2:59 pm
gmatmachoman wrote:TestLuv,

Once again such a wonderful post from you.

What makes me feel" mesmerised" is u r too good in deciphering the type of question be it LSAT or GMAT ones!!

Yeah this one was from LSAT....

Many Thx bro
Thanks again for the kind words, gmatmachoman!

You can tell this is an LSAT question because of the verbal "tics" that it uses. For example, in the last sentence, it throws in the intervening modifying phrase "although a small one" (the effect must have been small because the decrease was only 3 percent). This is verbal "filler" that one needs to cut through to really understand the decisive ideas in the argument. This is more of an LSAT than a GMAT test-maker tactic. So, that's why I suspected it was an LSAT question! Nonetheless, as I said in my post, this happens to be a very good representation of how strengthening/weakening questions often work on the GMAT.
Kaplan Teacher in Toronto

Legendary Member
Posts: 2326
Joined: Mon Jul 28, 2008 3:54 am
Thanked: 173 times
Followed by:2 members
GMAT Score:710

by gmatmachoman » Mon Dec 07, 2009 8:27 pm
TestLuv,

We just have no words to describe ur prowess !!! Just one word.. OMG .....we r Dumbstruck!!

Legendary Member
Posts: 1404
Joined: Tue May 20, 2008 6:55 pm
Thanked: 18 times
Followed by:2 members

by tanviet » Mon Mar 01, 2010 6:38 am
Testluv wrote:The correct answer must be choice D.

This is another explain the phenomenon or cause of the effect type of argument.

The first and second sentences describe a phenomenon, while the third sentence advances an explanation for that phenomenon.

The phenomenon: Smoking declined.

The author's explanation: The ads (and not the taxes!) caused the decrease in smoking. ("Clearly what was said in the advertisements had an effect"...)

Whenever the author makes an explain-the-phenomenon argument, he is assuming that there are no other explanations.

The way we strengthen an argument is by finding an answer choice that backs up the assumption.

How do we back up the assumption that there are no other explanations?

The most common way it will happen on the GMAT is to look for an answer choice that negates an alternative explanation.

A clear competing alternative explanation from the stimulus is the tax itself. (That is, instead of the ads' content causing the decline in smoking, it could have been the tax-fuelled increase in the costs of the cigarettes that caused the decline in smoking).

So, we would be looking for any choice that says "the tax was not the reason the smoking went down".

Choice D is a perfect match to our prediction. It tells us that the merchants absorbed the cigarette tax; therefore, the consumers who buy the cigarettes did not suffer, did not incur, the tax increase.

The fact that they did not have to pay more for the cigarettes and that they still decreased their smoking strengthens the author's argument that what was said in the ads caused the smoking decline.

Choice B does not strengthen the argument that smoking decreased because of the ads' content. Instead, it simply affirms the fact that smoking did decrease. In other words, choice B does not give us any new information; we already knew from the passage that smoking declined. For, that was the very phenomenon about which the author was advancing an explanation.

Although this looks like an LSAT question, it is a very fair and even a good representation of how strengthening/weakening commonly works on the GMAT. Many of the strengthen/weaken questions on the GMAT are of the "explain the phenomenon" variety. The quicker you are able to spot the form, and the better you know how to analyze the form, the more efficiently and confidently and easiliy you can answer a question like this. Therefore, according to me, a great idea would be to go through the CR section of the OG, explicitly pulling out those stn/wkn questions that use this form.

Legendary Member
Posts: 1404
Joined: Tue May 20, 2008 6:55 pm
Thanked: 18 times
Followed by:2 members

by tanviet » Mon Mar 01, 2010 6:41 am
Testluv wrote:The correct answer must be choice D.

This is another explain the phenomenon or cause of the effect type of argument.

The first and second sentences describe a phenomenon, while the third sentence advances an explanation for that phenomenon.

The phenomenon: Smoking declined.

The author's explanation: The ads (and not the taxes!) caused the decrease in smoking. ("Clearly what was said in the advertisements had an effect"...)

Whenever the author makes an explain-the-phenomenon argument, he is assuming that there are no other explanations.

The way we strengthen an argument is by finding an answer choice that backs up the assumption.

How do we back up the assumption that there are no other explanations?

The most common way it will happen on the GMAT is to look for an answer choice that negates an alternative explanation.

A clear competing alternative explanation from the stimulus is the tax itself. (That is, instead of the ads' content causing the decline in smoking, it could have been the tax-fuelled increase in the costs of the cigarettes that caused the decline in smoking).

So, we would be looking for any choice that says "the tax was not the reason the smoking went down".

Choice D is a perfect match to our prediction. It tells us that the merchants absorbed the cigarette tax; therefore, the consumers who buy the cigarettes did not suffer, did not incur, the tax increase.

The fact that they did not have to pay more for the cigarettes and that they still decreased their smoking strengthens the author's argument that what was said in the ads caused the smoking decline.

Choice B does not strengthen the argument that smoking decreased because of the ads' content. Instead, it simply affirms the fact that smoking did decrease. In other words, choice B does not give us any new information; we already knew from the passage that smoking declined. For, that was the very phenomenon about which the author was advancing an explanation.

Although this looks like an LSAT question, it is a very fair and even a good representation of how strengthening/weakening commonly works on the GMAT. Many of the strengthen/weaken questions on the GMAT are of the "explain the phenomenon" variety. The quicker you are able to spot the form, and the better you know how to analyze the form, the more efficiently and confidently and easiliy you can answer a question like this. Therefore, according to me, a great idea would be to go through the CR section of the OG, explicitly pulling out those stn/wkn questions that use this form.
Testluv,

I agree with you that D is perfect

but I see that A, B and C are new information which increase believing in conclusion and are correct answers

pls, explain, why A,B, C are wrong

should I study LSAT questions for gmat?

User avatar
Master | Next Rank: 500 Posts
Posts: 355
Joined: Thu Feb 19, 2009 12:42 pm
Thanked: 2 times
Followed by:1 members

by vineetbatra » Mon Mar 01, 2010 4:04 pm
Testluv wrote:


You can tell this is an LSAT question because of the verbal "tics" that it uses. For example, in the last sentence, it throws in the intervening modifying phrase "although a small one" (the effect must have been small because the decrease was only 3 percent). This is verbal "filler" that one needs to cut through to really understand the decisive ideas in the argument. This is more of an LSAT than a GMAT test-maker tactic. So, that's why I suspected it was an LSAT question! Nonetheless, as I said in my post, this happens to be a very good representation of how strengthening/weakening questions often work on the GMAT.
Test Luv a quick one on why B is not correct. The argument said that number of people decreased by 3%. But B says that the existing smokers smoke less but this does not prove that the number of people changed.

Do you think my observation is correct?

Vineet